Difference between revisions of "2006 AMC 12A Problems/Problem 4"
Line 1: | Line 1: | ||
== Problem == | == Problem == | ||
+ | |||
+ | A digital watch displays hours and minutes with AM and PM. What is the largest possible sum of the digits in the display? | ||
+ | |||
+ | <math> \mathrm{(A) \ } 17\qquad \mathrm{(B) \ } 19\qquad \mathrm{(C) \ } 21\qquad \mathrm{(D) \ } 22</math> | ||
+ | |||
+ | <math>\mathrm{(E) \ } 23</math> | ||
== Solution == | == Solution == |
Revision as of 22:41, 10 July 2006
Problem
A digital watch displays hours and minutes with AM and PM. What is the largest possible sum of the digits in the display?